Đến nội dung

Hình ảnh

[TOPIC] Hai bài toán mỗi ngày.


  • Please log in to reply
Chủ đề này có 173 trả lời

#61
tritanngo99

tritanngo99

    Đại úy

  • Điều hành viên THPT
  • 1644 Bài viết

Lời giải bài 45:

Giả sử $f:\mathbb{R}\to \mathbb{R}$ là hàm số thỏa mãn hệ thức của đề bài, nghĩa là:

$f(f(x-y))=f(x).f(y)-f(x)+f(y)-xy(1)$.

Đặt $f(0)=a$.

Thế $x=y=0$ vào $(1)$, ta được: $f(a)=a^2(2)$.

Thế $x=y$ vào $(1)$, với lưu ý tới $(2)$, ta được:

$(f(x))^2=x^2+a^2,\forall x\in \mathbb{R}(3)$.

Suy ra: $(f(x))^2=(f(-x))^2,\forall x\in \mathbb{R}(4)$.

Giả sử tồn tại $x_0\ne 0$ sao cho $f(x_0)=f(-x_0)$.

Thế $y=0$ vào $(1)$, được:

$f(f(x))=af(x)-f(x)+a,\forall x\in \mathbb{R}(5)$.

Thế $x=0,y=-x$ vào $(1)$, ta được:

$f(f(x))=af(-x)+f(-x)-a,\forall x\in \mathbb{R}(6)$.

Từ $(5),(6)$ suy ra:

$a(f(-x)-f(x))+f(x)+f(-x)=2a,\forall x\in \mathbb{R}(7)$.

Thế $x=x_0$ vào $(7)$, ta được: $f(x_0)=a(*)$.

Mặt khác, từ $(3)$ suy ra nếu $f(x_1)=f(x_2)$ thì $x_1^2=x_2^2$. Vì thế, từ $(*)$ suy ra $x_0=0$, trái với giả thiết $x_0\ne 0$.

Mâu thuẩn chứng tỏ $f(x)\ne f(-x),\forall x\ne 0$.

Do đó, từ $(4)$ suy ra: $f(x)=-f(-x),\forall x\ne 0(8)$.

Thế $(8)$ vào $(7)$, ta được:

   $a(f(x)-1)=0,\forall x\ne 0$.

Suy ra: $a=0$, vì nếu ngược lại ,tức là $a\ne 0$ thì $f(x)=1,\forall x\ne 0$, trái với $(8)$.

Do đó, từ $(3)$ có:$(f(x))^2=x^2,\forall x\in \mathbb{R}(9)$.

Giả sử tồn tại $x_0\ne 0$ sao cho $f(x_0)=x_0$. Khi đó theo $(5)$ ta phải có:

$x_0=f(x_0)=-f(f(x_0))=-f(x_0)=-x_0$.

Mâu thuẩn chứng tỏ $f(x)\ne x,\forall x\ne 0$.

vì vậy, từ $(9)$ ta được: $f(x)=-x,\forall x\in \mathbb{R}$.

Ngược lại, kiểm tra trực tiếp, ta thấy hàm số tìm được ở trên thỏa mãn các yêu cầu của đề bài.

Vậy hàm số $f(x)=-x,x\in \mathbb{R}$ là hàm số duy nhất cần tìm.

Lời giải bài 46:

Viết lại hệ thức của đề bài dưới dạng: $x!+y!=3^{n}.n!(1)$.

+ Giả sử $(x,y,n)$ là bộ số tự nhiên thỏa mãn $(1)$.

Dễ thấy phải có $n\ge 1$; không mất tính tổng quát, có thể giả sử: $x\le y$.

Khi đó xảy ra:

1) Trường hợp $1$: $x\le n$.

Ta có: $(1)\iff 1+\frac{y!}{x!}=3^n.\frac{n!}{x!}(2)$.

Suy ra: $1+\frac{y!}{x!}\equiv 0(\text{ mod } 3)$. Từ đây, lưu ý rằng tích của $3$ số nguyên liên tiếp là một số chia hết cho $3$ và $n\ge 1$.

Suy ra: $x<y\le x+2$.

+ Nếu $y=x+2$ thì từ $(2)$ ta có: $1+(x+1)(x+2)=3^{n}.\frac{n!}{x!}(3)$.

Với lưu ý rằng tích của hai số nguyên liên tiếp là một số chia hết cho $2$, từ $(3)$ suy ra $n\le x+1$.

+ Nếu $n=x$ thì từ $(3)$ ta được:

$1+(x+1)(x+2)=3^x$ hay $x^2+3x+3=3^x(4)$.

 Vì $x\ge 1$ nên từ $(4)$ suy ra $x\equiv 0(\text{ mod }3)$. Do đó, phải có: $x\ge 3$.

Khi đó, từ $(4)$ ta có: 

$-3=x^2+3x-3^x\equiv 0(\text{ mod 9})$.

Điều vô lí nhận được chứng tỏ $n\ne x$.

+ Với $n=x+1$, từ $(3)$ ta có: $1+(x+1)(x+2)=3^{n}(x+1)$.

Suy ra $x+1$ là ước nguyên dương của $1$. Do đó $x=0$, kéo theo $y=2$ và $n=1$.

+Nếu $y=x+1$ thì từ $(2)$ ta có: $x+2=3^n.\frac{n!}{x!}(5)$.

 Với chú ý rằng: $n\ge 1,$ từ $(5)$ suy ra $x\ge 1$. Do đó: $x+2\equiv 1(\text{ mod }(x+1))$.

Vì thế, từ $(5)$ suy ra $n=x$. Khi đó, từ $(5)$ ta có: $x+2=3^{x}(6)$.

Dễ thấy, có duy nhất giá trị $x=1$ thỏa mãn $(6)$, ta được bộ số $(x=1,y=2,n=1)$.

Vậy trong trường hợp này, nếu bộ số tự nhiên $(x,y,n)$ thỏa mãn $(1)$ thì $(x,y,n)=(0,2,1)$ hoặc $(x,y,n)=(1,2,1)$.

2) Trường hợp $2$: $x>n$.

Ta có: $(1)\iff \frac{x!}{n!}+\frac{y!}{n!}=3^{n}(7)$.

Với lưu ý rằng $n+1$ và $n+2$ không thể đồng thời là lũy thừa của $3$, từ $(7)$ suy ra $x=n+1$. 

Khi đó, từ $(2)$ suy ra: $n+1+\frac{y!}{n!}=3^n(8)$.

Vì $y\ge x$ nên $y\ge n+1$. Đặt $A=\frac{y!}{(n+1)!}$. Khi đó có thể viết hệ thức $(8)$ dưới dạng: $(n+1)(1+A)=3^{n}(9)$.

Rõ ràng, nếu $y\ge n+4$ thì $A\equiv 0(\text{ mod 3})$; vì thế, $A+1$ không thể là lũy thừa của $3$. Do đó, từ $(9)$ suy ra: $y\le n+3.$.

+Nếu $y=n+3$ thì $A=(n+2)(n+3)$. Do đó,  từ $(9)$ ta có: $(n+1)(1+(n+2)(n+3))=3^{n}$, hay $(n+2)^3-1=3^n(10)$.

Suy ra $n>2$ và $n+2\equiv 1(\text{ mod 3})$.

Đặt $n+2=3k+1,k\ge 2$. khi đó, $(10)$ trở thành :$9k(3k^2+3k+1)=3^{3k-1}$.

Suy ra $3k^2+3k+1$ là một lũy thừa của $3$. Điều vô lý vừa nhận được chứng tỏ $y\ne n+3$.

+ Nếu $y=n+2$ thì $A=n+2$. Do đó, từ $(9)$ , ta có: $(n+1)(n+3)=3^{n}(11)$.

Vì $n+1$ và $n+3$ không thể đồng thời là các lũy thừa của $3$ nên không tồn tại $n$ thỏa mãn $(11)$. Do đó $y\ne n+2$.

+ Với $y=n+1$, ta có $A=1$. Do đó, từ $(9)$ ta được: $2(n+1)=3^n$. Rõ ràng, không tồn tại $n$ thỏa mãn hệ thức vừa nêu. Do vậy $y\ne n+1$.

Như vậy, nếu bộ số tự nhiên $(x,y,n)$ với $x\le y$, thỏa mãn $(1)$ thì không thể có $x>n$.

+ Tóm lại, nếu $(x,y,n)$ là bộ số tự nhiên thỏa mãn $(1)$ thì $(x,y,n)=(0,2,1)$ hoặc $(x,y,n)=(2,0,1)$ hoặc $(x,y,n)=(1,2,1)$ hoặc $(x,y,n)=(2,1,1)$.

Ngược lại, kiểm tra trực tiếp, dễ thấy tất cả bốn bộ số vừa nêu đều thỏa mãn $(1)$.

Vậy bốn bộ số nêu trên là tất cả các bộ số cần tìm theo yêu cầu bài toán.


Bài viết đã được chỉnh sửa nội dung bởi tritanngo99: 02-09-2018 - 04:52


#62
tritanngo99

tritanngo99

    Đại úy

  • Điều hành viên THPT
  • 1644 Bài viết

Bài 47: Cho $P=(n+1)^7-n^7-1(n\in \mathbb{N})$. Chứng minh rằng có vô hạn số tự nhiên $n$ để $P$ là một số chính phương.

Bài 48: Cho $X$ là tập hợp con của tập hợp $\left\{1,2,3,...,2010\right\}$ thỏa mãn đồng thời các điều kiện sau đây:

1) $|X|=62$.

2) Với mọi $x\in X$ đều tồn tại $a,b\in X\cup \text{{0;2011}}$ ,($a$ và $b$ đều khác $x$) sao cho $x=\frac{a+b}{2}$.

Chứng minh rằng có hai phần tử $x,y$ của $X$ sao cho $|x-y|\ge 11$ và $\frac{x+y}{2}$ không thuộc $X$.

 


#63
Duy Thai2002

Duy Thai2002

    Sĩ quan

  • Thành viên
  • 433 Bài viết
bài 47 có 1 hướng tiếp cận là sử dụng phương trình pell. Lời giải sẽ gửi sau.

Sự khác biệt giữa thiên tài và kẻ ngu dốt là ở chỗ thiên tài luôn có giới hạn.


#64
Duy Thai2002

Duy Thai2002

    Sĩ quan

  • Thành viên
  • 433 Bài viết

Bài 47. Ta có: $(n+1)^{7}-n^{7}-1=7n(n+1)(n^{2}+n+1)^{2}$

Ta sẽ chứng minh có vô số số tự nhiên $n$ để $7n(n+1)$ luôn là số chính phương.

Điều này $<=>$ pt: $n(n+1)=7k^{2}$ có vô số nghiệm

Giả sử $k=x.y$ Ta xét trường hợp sau:

$\left\{\begin{matrix}n=7y^{2} & \\n+1=x^{2} & \end{matrix}\right.$

$=> x^{2}-7y^{2}=1$. Ta nhận thấy đây là phương trình Pell loại 1 nên có vô số nghiệm $x,y$ nguyên dương.

Do đó, ứng với mọi $x,y$ là nghiệm cuả phương trình Pell thì sẽ có $n$ tương ứng sao cho $7n(n+1)$  là số chính phương. Mà có vô số $x,y$ như vậy nên sẽ có vô số $n$.

Vậy ta có đpcm.


Bài viết đã được chỉnh sửa nội dung bởi Duy Thai2002: 03-09-2018 - 06:29

Sự khác biệt giữa thiên tài và kẻ ngu dốt là ở chỗ thiên tài luôn có giới hạn.


#65
tritanngo99

tritanngo99

    Đại úy

  • Điều hành viên THPT
  • 1644 Bài viết

Lời giải bài 47:

Ta có: $P=(n+1)^7-n^7-1=7n(n+1)(n^2+n+1)^2$.

Thành thử ta chỉ cần chỉ ra có vô hạn số tự nhiên $n$ để $7n(n+1)$ là số chính phương hay để $n(n+1)=7y^2(y\in\mathbb{N^*})$.

Ta có:$n(n+1)=7y^2\iff (2n+1)^2-28y^2=1(1)$.

Đặt $x=2n+1$ ta được: $x^2-28y^2=1(2)$.

Ta đã biết phương trình $(2)$(gọi là phương trình Pell có vô số nghiệm nguyên dương).

Ngược lại, nếu $(x;y)$ là nghiệm của $PT(2)$ thì $x$ lẻ, do đó $n=\frac{x-1}{2}$ thỏa mãn $(1)$. Do đó với mỗi nghiệm nguyên dương của $(2)$ thì ứng với một giá trị $n=\frac{x-1}{2}$ để $n(n+1)=7y^2\iff P$ là số chính phương. Vậy bài toán được giải xong.

Lời giải bài 48:

Giả sử $X$ có $62$ phần tử $x_1,x_2,...,x_{62}$ đã được sắp thứ tự $x_1<x_2<...<x_{62}$. Ta chứng minh bài toán bằng phương pháp phản chứng.

Giả sử không tồn tại hai phần tử $x,y$ của $X$ sao cho $|x-y|\ge 11$ và $\frac{x+y}{2}$ không thuộc $X$. Ta sẽ chứng minh rằng các phần tử của $X$ đều cùng tính chẵn lẻ.

Vì $X\subset\left\{1,2,3,...,2010\right\}$ nên $x_{i+1}\ge x_{i}+1$ với $i=1,2,3,...,61$. Do đó $x_{62}-x_k\ge 11$ với $k=1,2,...,51$. Suy ra $x_{k}(k=1,2,...,51)$ cùng tính chẵn lẻ với $x_{62}$, vì nếu không như vậy, tức tồn tại $x_k$ khác tính chẵn lẻ với $x_{62}$ thì $\frac{x_k+x_{62}}{2}\in X$, trái với giả thiết. Lập luận tương tự, ta thấy $x_l,l=12,13,...,62$ cùng tính chẵn lẻ với $x_1$. Từ đó suy ra mọi $x_1,x_2,...,x_{62}$ cùng tính chẵn lẻ. Tiếp theo với giả thiết 2) của bài toán thì tồn tại $a,b\in X\subset\left\{0;2011\right\}(a\ne b)$ sao cho $x_{62}=\frac{a+b}{2}$. Vì $x_{62}>x_i,i=1,2,...,61$ nên tồn tại $x_p$ với $p=1,2,...,61$ sao cho $x_{62}=\frac{x_{p}+2011}{2}$ kéo theo $x_p$ là một số lẻ. Tương tự, do $x_i<x_1$,với $i=2,3,...,61$ nên tồn tại $x_q$ với $q=2,3,...,61$ sao cho $x_1=\frac{x_q+0}{2}$, kéo theo $x_q$ là một số chẵn. Như vậy, $x_p$ và $x_q$ là hai phần tử của $X$ khác tính chẵn lẻ, trái với kết quả khẳng định trên, tức điều giả sử là sai.

Kết luận: Vậy, có hai phần tử $x,y$ của $X$ sao cho $|x-y|\ge 11$ và $\frac{x+y}{2}$ không thuộc $X$.


Bài viết đã được chỉnh sửa nội dung bởi tritanngo99: 03-09-2018 - 20:14


#66
tritanngo99

tritanngo99

    Đại úy

  • Điều hành viên THPT
  • 1644 Bài viết

Bài 49: Cho $a,b,n$ là những số nguyên dương, $b>1$ và a chia hết cho $b^n-1$. Chứng minh rằng khi biểu diễn số $a$ trong hệ cơ số $b$ thì được số chứa ít nhất $n$ chữ số khác $0$.

Bài 50: Xét tứ giác $ABCD$ có các đường chéo cắt nhau ở $K$ và $\angle{ABC}=\angle{ADC}=90^0;AC=AB+AD$. Chứng minh rằng bán kính đường tròn nội tiếp các tam giác $ABK,ADK$ bằng nhau.

 


#67
Hr MiSu

Hr MiSu

    Thượng sĩ

  • Thành viên
  • 206 Bài viết

Bài 49: Cho $a,b,n$ là những số nguyên dương, $b>1$ và a chia hết cho $b^n-1$. Chứng minh rằng khi biểu diễn số $a$ trong hệ cơ số $b$ thì được số chứa ít nhất $n$ chữ số khác $0$.

Bài này phát biểu đẹp, ko biết mình giải đúng ko nữa:

Capture664c9a4da0b400d2.png


s2_PADY_s2

Hope is a good thing, maybe the best thing, and no good thing ever dies


#68
tritanngo99

tritanngo99

    Đại úy

  • Điều hành viên THPT
  • 1644 Bài viết

Lời giải bài 49:

Gọi $\mathscr{A}=\left\{a\in \mathbb{N^*}|a\vdots b^{n}-1\right\}$. Giả sử $s(a)$ là số chữ số khác $0$ của $a\in \mathscr{A}$ trong biểu diễn cơ số $b$. Đặt $s=min(s(a))$ và $\mathscr{H}=\left\{a\in \mathscr{A}:s(a)=s\right\}$. Trong tập $\mathscr{H}$ ta chọn số $A$ có tổng các chữ số trong biểu diễn cơ số $b$ bé nhất.

Giả sử $A=\sum\limits_{i=1}^{s}a_ib^{n_i}$ với $n_1>n_2>...>n_s$ và $a_i\in \left\{1,2,...,b-1\right\}$. 

Ta chứng minh $n_i\not\equiv n_j(\text{ mod n})$ với $i\ne j$. Thật vậy, giả sử $n_i\equiv n_j(\text{ mod n})$ ( với $1\le i<j\le s$).

Đặt $r\equiv n_i\equiv n_j(\text{ mod n})(0\le r\le n-1)$.

Xét số $B=A-a_ib^{n_i}-a_jb^{n_j}+(a_i+a_j)b^{nn_1+r}$.

Ta có:

$A-B=a_i(b^{n_i}-b^{nn_1+r})+a_j(b^{n_j}-b^{nn_1+r})$.

Ta có:

$A-B=a_i(b^{n_i}-b^{nn_1+r})+a_j(b^{n_j}-b^{nn_1+r})\vdots b^{n}-1$.

Do đó $B\in \mathscr{A}$.

+ Nếu $a_i+a_j\le b-1$ thì số chữ số của $B$ là $s-1$ mâu thuẩn với cách chọn.

+ Nếu $b\le a_i+a_j<2b$. Đặt $a_i+a_j=b+q(0\le q<b)$. Khi đó $b$ có biểu diễn:

$B=b^{nn_1+r+1}+qb^{nn_1+r}+a_1b^{n_1}+...+a_{i-1}b^{n_{i-1}}+...+a_{i+1}b^{n_{i+1}}+...+a_{j-1}b^{n_{j-1}}+a_{j+1}b^{n_{j+1}}+...+a_sb^{n_s}$.

Khi đó tổng các chữ số của $B$ là :

$\sum\limits_{i=1}^{s}a_i-(a_i+a_j)+1+q=\sum\limits_{i=1}^{s}a_i+1-b<\sum\limits_{i=1}^{s}a_i$.

Điều này trái với cách chọn $A$. Vậy $n_i\not\equiv n_j(\text{ mod n})(i\ne j)$. Từ đó phải có : $s\le n$. Ta chứng minh: $s=n$.

Thật vậy, giả sử $s<n$. Đặt $n_i\equiv r_i(\text{ mod n})$ với $i=1,2,...,s$. Xét số $C=\sum\limits_{i=1}^{s}a_ib^{r_i}$. Ta có:

$A-C=\sum\limits_{i=1}^{s}a_i(b_i^{n_i}-b_j^{n_j})\vdots b^{n}-1$, do đó $C\in \mathscr{A}$.

Mặt khác do $0<a_i\le b-1,s\le n-1$ nên $0<C\le (b-1)b+(b-1)b^2+...+(b-1)b^{n-1}$.

$=b^n-b<b^n-1$. Mâu thuẩn.

Vậy $s=n$. Bài toán được chứng minh.

Lời giải bài 50:cvcc.jpg

Gọi $O$ là trung điểm của $AC$. Dễ thấy tứ giác $ABCD$ nội tiếp đường tròn tâm $O$, đường kính $AC$. 

Trên $AC$ lấy các điểm $E,F$ sao cho $AE=AB;AF=AD$. Từ giả thiết : $AC=AB+AD$, dễ thấy $OE=OF$.

Lấy $M$ thuộc đoạn $KB$ sao cho $AM$ là phân giác của góc $\angle{AKB}(1)$.

Lấy $P$ thuộc tia $AM$ sao cho $OP\parallel BD$. Lấy $Q$ trên tia đối của tia $OP$ sao cho $OQ=OP$.

Dễ thấy $BPQD$ là hình thang cân; $EPFQ$ là hình bình hành. Suy ra $DQ=BP=EP=FQ$. Do đó các tam giác $ADQ,AFQ$ bằng nhau(c.c.c)

Từ đó $AQ$ là phân giác của góc $\angle{KAD}(2)$.

Gọi $N$ là giao điểm của $AQ$ và $KD$. Theo định lý Ta-let , ta có: $\frac{KM}{OP}=\frac{KN}{OQ}$. Từ đó, với chú ý rằng $OP=OQ$, suy ra $KM=KN(3)$.

Từ $(1),(2),(3)$ theo tính chất của đường phân giác, ta có:

$\frac{KA.KB}{KA+KB}=\frac{KA.KD}{KA+AD}\implies \frac{KB}{KD}=\frac{KA+AB}{KD+AD}$.

$\implies \frac{KB}{KD}=\frac{KA+AB+KB}{KD+AD+KD}(4)$.

Gọi $S_1,S_2;p_1,p_2;r_1,r_2$ theo thứ tự là diện tích, nửa chu vi, bán kính đường tròn nội tiếp của các tam giác $AKB,AKD$.

Từ $(4)$ suy ra: $\frac{S_1}{S_2}=\frac{2p_1}{2p_2}\implies \frac{p_1r_1}{p_2r_2}=\frac{2p_1}{2p_2}$.

$\implies r_1=r_2$

 

Bài viết đã được chỉnh sửa nội dung bởi tritanngo99: 04-09-2018 - 05:12


#69
tritanngo99

tritanngo99

    Đại úy

  • Điều hành viên THPT
  • 1644 Bài viết

Bài 51: Cho $n$ là một số nguyên dương và $2n+2$ số thực $a,b,a_1,a_2,...,a_n,b_1,b_2,...,b_n$ với $a_i\ne 0(i=1,2,...,n)$ sao cho hàm số $F(x)=\sum\limits_{i=1}^{n}\sqrt{a_ix+b_i}-(ax+b)$ có tính chất:

Tồn tại hai số thực phân biệt $\alpha,\beta$ sao cho $F(\alpha)=F(\beta)=0$.

Chứng minh rằng: $\alpha$ và $\beta$ là tất cả các nghiệm thực của phương trình $F(x)=0$.

Bài 52: Cho tập hợp $X=\left\{0,1,2,...,n\right\}$ với $n$ là số nguyên dương. Với $x=(x_1,x_2,x_3),y=(y_1,y_2,y_3)$ thuộc $X^3$, ta nói $xRy$ nếu $x_k\le y_k$ với $k=1,2,3$. Hãy xác định số nguyên dương nhỏ nhất $p$ sao cho với bất kì tập con $A$ có $p$ phần tử nào của $X^3$ đều có tính chất: Nếu $x,y$ thuộc $A$ thì $xRy$ hoặc $yRx$



#70
hozymary

hozymary

    Binh nhì

  • Thành viên mới
  • 19 Bài viết

Bài 51: Cho $n$ là một số nguyên dương và $2n+2$ số thực $a,b,a_1,a_2,...,a_n,b_1,b_2,...,b_n$ với $a_i\ne 0(i=1,2,...,n)$ sao cho hàm số $F(x)=\sum\limits_{i=1}^{n}\sqrt{a_ix+b_i}-(ax+b)$ có tính chất:

Tồn tại hai số thực phân biệt $\alpha,\beta$ sao cho $F(\alpha)=F(\beta)=0$.

Chứng minh rằng: $\alpha$ và $\beta$ là tất cả các nghiệm thực của phương trình $F(x)=0$.

Giả sử PT $F(x)=0$ có nghiệm $\gamma \notin \{\alpha, \beta\}$. Ta có

$$F(a)=F(b) \Leftrightarrow \sum \sqrt{a_i \alpha+b_i} - \sum \sqrt{a_i \beta + b_i}- a(\alpha - \beta) = 0$$

$$\quad \quad \Leftrightarrow \sum \frac{a_i(\alpha-\beta)}{\sqrt{a_i\alpha+b_i}+\sqrt{a_i\beta+b_i}}=a(\alpha-\beta)$$

$$\quad \quad \quad \ \ \Leftrightarrow \sum \frac{a_i}{\sqrt{a_i\alpha+b_i}+\sqrt{a_i\beta+b_i}}=a \ (\text{do } \alpha \neq \beta)$$

CMTT ta được $\sum \frac{a_i}{\sqrt{a_i\alpha+b_i}+\sqrt{a_i\gamma+b_i}}=a$

Lấy hai đẳng thức trên trừ nhau ta được $\sum a_i\left(\frac{1}{\sqrt{a_i\alpha+b_i}+\sqrt{a_i\beta+b_i}}-\frac{1}{\sqrt{a_i\alpha+b_i} +\sqrt{a_i\gamma+b_i}}\right)=0$

$\Leftrightarrow \sum a_i\left[\frac{\sqrt{a_i\gamma+b_i}-\sqrt{a_i\beta+b_i}}{\left(\sqrt{a_i\alpha+b_i}+\sqrt{a_i\beta+b_i}\right) \left(\sqrt{a_i\alpha+b_i} +\sqrt{a_i\gamma+b_i}\right)}\right]=0\\ \Leftrightarrow \sum \frac{a_i^2}{\left(\sqrt{a_i\alpha+b_i}+\sqrt{a_i\beta+b_i}\right) \left(\sqrt{a_i\beta+b_i}+\sqrt{a_i\gamma+b_i}\right) \left(\sqrt{a_i\gamma+b_i}+\sqrt{a_i\alpha+b_i}\right)} = 0$
Hiển nhiên điều trên vô lý với $a_i \neq 0$, suy ra điều phải chứng minh.



#71
tritanngo99

tritanngo99

    Đại úy

  • Điều hành viên THPT
  • 1644 Bài viết

Lời giải bài 51:

Giả sử $\exists \gamma$ với $\gamma\ne \alpha,\gamma\ne \beta$ sao cho $F(\gamma)=0$.

Ta có: $F(\alpha)=F(\gamma)$.

$\iff \sum\limits_{i=1}^{n}(\sqrt{a_i\alpha+b_i}-\sqrt{a_i\gamma+b_i})=a(\alpha-\gamma)$.

$\iff \sum\limits_{i=1}^{n}\frac{a_i(\alpha-\gamma)}{\sqrt{a_i\alpha+b_i}+\sqrt{a_i\gamma+b_i}}=a(\alpha-\gamma)$.

$\iff \sum\limits_{i=1}^{n}\frac{a_i}{\sqrt{a_i\alpha+b_i}+\sqrt{a_i\gamma+b_i}}=a$( do $\alpha\ne \gamma$).(1).

Tương tự: $\sum\limits_{i=1}^{n}\frac{a_i}{\sqrt{a_i\beta+b_i}+\sqrt{a_i\gamma+b_i}}=a$.(2)

Từ $(1)$ và $(2)$ suy ra:

$0=\sum\limits_{i=1}^{n}(\frac{1}{\sqrt{a_i\alpha+b_i}+\sqrt{a_i\gamma+b_i}}-\frac{1}{\sqrt{a_i\beta+b_i}+\sqrt{a_i\gamma+b_i}})$.

$\iff 0=\sum\limits_{i=1}^{n}\frac{a_i(\sqrt{a_i\beta+b_i}-\sqrt{a_i\alpha+b_i})}{(\sqrt{a_i\alpha+b_i}+\sqrt{a_i\gamma+b_i})(\sqrt{a_i\beta+b_i}+\sqrt{a_i\gamma+b_i})}$

$\iff 0=\sum\limits_{i=1}^{n}\frac{a_i^2(\beta-\alpha)}{(\sqrt{a_i\alpha+b_i}+\sqrt{a_i\gamma+b_i})(\sqrt{a_i\beta+b_i}+\sqrt{a_i\gamma+b_i})}.\frac{1}{\sqrt{a_i\beta+b_i}+\sqrt{a_i\alpha+b_i}}$.

Vì $\beta\ne \alpha$ nên ta suy ra:

$0=\sum\limits_{i=1}^{n}\frac{a_i^2}{(\sqrt{a_i\alpha+b_i}+\sqrt{a_i\gamma+b_i})(\sqrt{a_i\beta+b_i}+\sqrt{a_i\gamma+b_i})}.\frac{1}{\sqrt{a_i\beta+b_i}+\sqrt{a_i\alpha+b_i}}$.

Do $a_i\ne 0$ nên ta có mâu thuẩn. Vậy $\alpha,\beta$ là tất cả các nghiệm của $PT:F(x)=0$.

Lời giải bài 52:

Với mỗi $x=(x_1,x_2,x_3)$, đặt $s(x)=x_1+x_2+x_3$.

Ta nói tập $Y\subset X^3$ là tập loại $T$ nếu với mỗi $x,y\in Y$ ta có $xRy$ hay $yRx$.

Ta nói tập $Y\subset X^3$ là loại tập $F$ nếu không tồn tại $x,y\in Y$ nào để $xRy$.

Với mỗi $k\in \left\{0,1,2,...,3n\right\}$, đặt $A_k=\left\{a\in Y|s(a)=k\right\}$.

Dễ thấy $A_k$ là tập loại $F$. Ngoài ra:

$\left\{\begin{array}{I}|A_k|\le |A_{k+1}|, \text{ với } k=1,2,...,[\frac{3n}{2}]\\ |A_k|=|A_{3n-k}| \end{array}\right.$

Đặt $r=[\frac{n}{2}]$. Khi đó trong các tập $A_k$, tập $A_{n+r}$ có số phần tử lớn nhất. Kí hiệu $q=|A_{n+r}|$.

Bổ đề: Phương trình $x_1+x_2+...+x_s=t(t\ge s)$ có đúng $C_{s+t-1}^{s-1}$ nghiệm tự nhiên.

Sử dụng bổ đề trên ta tính được: $q=C_{n+r+2}^{2}-3C_{r+1}^2$.

Với mỗi $k=0,1,...,n$, ta đặt $B_k=\left\{(k,a)|n-k\ge a\in X\right\}\cup\left\{(a,n-k),k\le a\in X\right\}$.

+ Với $k=0,1,2,...,r$ và $j=0,1,2,...,n$, đặt $B_{kj}=\left\{(a,b,j)|(a,b)\in B_k\right\}$, có $(n+1)(r+1)$ tập $B_{kj}$.

+ Với $k=r+1,...,n$ và $(a,b)\in A_k$, đặt $A_k(a,b)=\left\{(a,b,k)|k\in X\right\}$, ta được $(n-r)^2$ tập như vậy.

Số các tập $B_{kj};A_k(a,b)$ là: $d=(n-r)^2+(n+1)(r+1)$.

Dễ thấy $d=q$. Kí hiệu các tập đã nêu trên là $(X_i)_{i=1}^q$. Rõ ràng các $X_i$ có tính chất $T$ và $(X_i)_{i=1}^{q}$ là một phân hoạch của $X^3$.

Với mỗi tập $A\subset X^3$ mà $|A|\ge q+1$ phần tử thì theo nguyên lí Dirichlet với mỗi $x,y\in A$ tồn tại $X_i$ sao cho $x,y\in X_i$. Do đó $xRy$ hoặc $yRx$. Thành thử $p=q+1$ chính là số cần tìm( vì $A_{n+r}$ có tính chất $F$ và $|A_{n+r}|=q$).

Đáp số: $p=C_{n+r+2}^{2}-3C_{r+1}^{2}+1$ với $r=[\frac{n}{2}]$

 


Bài viết đã được chỉnh sửa nội dung bởi tritanngo99: 05-09-2018 - 05:13


#72
tritanngo99

tritanngo99

    Đại úy

  • Điều hành viên THPT
  • 1644 Bài viết

Bài 53: Trong mặt phẳng cho $4$ điểm phân biệt $A_1,A_2,A_3,A_4$ trong đó không có $3$ điểm nào thẳng hàng. Gọi $A_{ij},A_{ik},A_{ih}$ là hình chiếu vuông góc của $A_i(i=1,2,3,4)$ trên các cạnh $A_kA_h,A_hA_j,A_jA_k$ của tam giác $A_jA_kA_h$. Gọi $C_i$ là đường tròn( hoặc đường thẳng ) qua ba điểm $A_{ij},A_{ik},A_{ih}$ và $E_i$ là đường tròn Ơle của tam giác $A_jA_kA_h$. Chứng minh rằng các đường tròn $C_i$ và $E_i$ cùng đi qua một điểm.

Bài 54: Tìm tất cả các hàm số tăng thực sự $f:\mathbb{N^*}\to \mathbb{N^*}$ thỏa mãn đồng thời hai điều kiện sau:

a) $f(2n)=f(n)+n$ với mọi $n\in \mathbb{N^*}$.

b) Nếu $f(n)$ là số chính phương thì $n$ là số chính phương.

 


#73
tritanngo99

tritanngo99

    Đại úy

  • Điều hành viên THPT
  • 1644 Bài viết

Lời giải bài 53: Để giải quyết trọn vẹn bài toán này ta cần đến khái niệm góc định hướng $(x,y)$ giữa hai đường thẳng $x$ và $y$ theo modun $\pi$.

Bổ đề 1: Cho hình bình hành $PQRS$ với $O$ là giao điểm hai đường chéo, $M$ là điểm bất kì, $N$ là điểm đối xứng của $M$ qua $O$. Khi đó các đường tròn ngoại tiếp các tam giác $PRM,SQM,PSN,RQN$ cùng đi qua một điểm.

Chứng minh: Gọi $T$ là giao điểm của các đường tròn ngoại tiếp hai tam giác $PRM,SQM(T\ne M,\text{ nếu }T\equiv M \text{ thì coi }MT \text{ là tiếp tuyến chung})$. Ta thấy: $(TP,TS)\equiv (TP,TM)+(TM,TS)\equiv (RP,RM)+(QM,QS)\equiv (SQ,SN)+(PN,PR)\equiv (NP,NS)(\text{ mod }\pi)$ .

Suy ra: $T$ thuộc đường tròn ngoại tiếp tam giác $PSN$. Tương tự, $T$ thuộc đường tròn ngoại tiếp tam giác $RQN$. Bổ đề $1$ đã được chứng minh.

Bổ đề 2: Cho tứ giác $ABCD$. Chứng minh rằng: Đường tròn Ơle của các tam giác $BCD,CDA,DAB,ABC$ cùng đi qua một điểm.

Chứng minh: 

Gọi $P,Q,N,M,R,S$ lần lượt là trung điểm của các đoạn $AB,CD,AC,DB,AD,BC$. Dễ thấy $MN,PQ,RS$ đồng quy tại trung điểm $O$ của mỗi đường.

Xét hình bình hành $PRQS$, theo bổ đề 1, các đường tròn ngoại tiếp các tam giác $QSM,QRN,PRM,PSN$ cùng đi qua một điểm, đó là đường tròn Ơle của các tam giác $BCD,CDA,DAB,ABC$ tương ứng. Bổ đề 2 được chứng minh.

Bổ đề 3: Cho bốn điểm $A_i(i=1,2,3,4)$ cùng nằm trên một đường tròn. Gọi $T_i$ là tam giác xác định bởi ba trong bốn điểm trên trừ điểm $A_i$. Chứng minh bốn đường tròn Ơle của $T_i$, bốn đường thẳng Simson của điểm $A_i$ đối với $T_i$ cùng qua một điểm.

( Đường thẳng Simson của điểm $A_i$ đối với tam giác $T_i$ là đường thẳng đi qua chân đường vuông góc kẻ từ $A_i$ đến các cạnh của $T_i$).

Chứng minh:

Gọi $G$ là trọng tâm của tứ điểm $\left\{A_1,A_2,A_3,A_4\right\}$ và $H_i$ là trực tâm của tam giác $(T_i)$ có các đỉnh $A_j,A_k,A_l$; với $i=1,2,3,4$ và $\left\{i,j,k,l\right\}=\left\{1,2,3,4\right\}$. Thế thì ta có:

$\vec{OH_i}=\vec{OA_j}+\vec{OA_k}+\vec{OA_l};(1)$.

do đó: $\vec{OA_i}+\vec{OH_i}=\vec{OA_1}+\vec{OA_2}+\vec{OA_3}+\vec{OA_4} (\forall i)=4\vec{OG}=2\vec{OO'}(2)$

trong đó $O'=Đ_G(O)$, đối xứng của $O$ qua $G$. Hệ thức vector $(2)$ chứng tỏ rằng đoạn thẳng $A_iH_i$ nhận điểm $O'$ (đối xứng với $O$ qua $G$) làm trung điểm; nói khác đi là bốn đoạn thẳng $A_iH_i$ đồng quy tại trung điểm chung $O'$ và ta có:

$\vec{H_iO'}=\frac{1}{2}\vec{H_iA_i}(3)$.

Ta lại biết rằng tâm $\omega_i$ của đường tròn Ơle của tam giác $T_i(A_jA_kA_l)$ là trung điểm của đoạn thẳng $OH_i$ nối tâm $O$ đường tròn ngoại tiếp với trực tâm $H_i$ của tam giác $T_i$, nghĩa là:

$\vec{H_i\omega_i}=\frac{1}{2}\vec{H_iO}(4)$.

Từ $(3),(4)$ suy ra: phép vị tự $V_{H_i}^{k}=\frac{1}{2}$ tâm $H_i$, tỷ số vị tự $k=\frac{1}{2}$ biến đường tròn $(O)$ thành đường tròn $(\omega_i);$ mà $(O)$ đi qua $\left\{A_i\right\}$ nên $(\omega_i)$ đi qua $(O')$. Vậy:

Bốn đường tròn Ơle(\omega_i) của các tam giác $(T_i)$ cùng đi qua $O'=Đ_G(O)$.

Bây giờ, gọi $B_i,B_i'$ và $B"$ là các điểm đối xứng của điểm $A_i$ lần lượt qua các đường thẳng $A_jA_k,A_kA_l,A_lA_j$ chứa các cạnh của tam giác (T_i), thế thì các điểm này thẳng hàng trên đường thẳng $Steiner(\delta_i)$ vị tự của đường thẳng Simson $\Delta_i$ của $A_i$ đối với tam giác $(T_i)$. Để chứng minh $\Delta_i$ đi qua $O'$ ta chứng minh $\delta_i$ đi qua $H_i$. Thật vậy, trực tâm $H_i$ của $(T_i)$, điểm $B_i$, đối xứng của $A_i$ qua $(A_jA_k)$ và các điểm $A_j,A_k$ cùng nằm trên đường tròn đối xứng với đường tròn $(O)$ qua đường thẳng $A_jA_k$. Hoàn toàn tương tự, $H_i$ và điểm $B_i',$ đối xứng của $A_i$ qua $(A_kA_l)$ cùng nằm trên đường tròn đối xứng với đường tròn $(O)$ qua đường thẳng $A_kA_l$.

Từ đó, sử dụng góc có hướng giữa hai đường thẳng, ta được $(H_iB_i,H_iA_k)=(A_kB_i,A_jA_k)=(A_jA_k,A_jA_i)(\text{ mod }\pi)(5)$.

  và: $(H_iB_i',H_iA_k)=(A_lB_i',A_lA_k)=(A_lA_k,A_lA_i)(\text{ mod }\pi)(6)$.

Lại vì $A_i,A_j,A_k,A_l$ cùng nằm trên đường tròn $(O)$ nên có:

$(A_jA_k,A_jA_i)=(A_lA_k,A_lA_i)(\text{ mod }\pi)(7)$.

Bởi vậy ta được: 

$(H_iB_i, H_iA_k)=(H_iB_i',H_iA_k)(\text{ mod }\pi)(8)$.

 Từ đó suy ra hai đường thẳng $(H_iB_i)$ và $(H_iB_i')$ phải trùng nhau; nói khác đi là $H_i$ nằm trên đường thẳng Steiner $\delta_i$ chứa các điểm $B_i,B_i',B_i''$. Đường thẳng Simson $\Delta_i$ của điểm $A_i$ đối với $(T_i)$ là ảnh của đường thẳng Steiner $\delta_i$ trong phép vị tự $V_{A_i}^{k}=\frac{1}{2}$ tâm $A_i,k=\frac{1}{2};$ mà $\delta_i$ đi qua $H_i$ nên $\Delta_i$ đi qua điểm $O'=V_{A_i}^{k}(H_i)$, ảnh của $H_i$ trong phép vị tự đó. Vậy bốn đường thẳng Simson $\Delta$ cùng đi qua $O'$.

Vậy bổ đề được chứng minh. 

Trở lại bài toán của ta.

Nếu $A_1,A_2,A_3,A_4$ cùng nằm trên một đường tròn thì $C_i(i=1,2,3,4)$ là các đường thẳng và $C_i,E_i(i=1,2,3,4)$ cùng đi qua một điểm (bổ đề 3).

Nếu $A_1,A_2,A_3,A_4$ không cùng nằm trên một đường tròn thì $C_i(i=1,2,3)$ là các đường tròn được chứng minh như sau:

Theo bổ đề 2, đường tròn Ơle của các tam giác $A_2A_3A_4,A_3A_4A_1,A_4A_1A_2,A_1A_2A_3$ cùng đi qua một điểm, gọi điểm đó là $T$. Ta sẽ chứng minh đường tròn ngoại tiếp các tam giác $A_{ij}A_{ik}A_{ih}(i=1,2,3,4)$ cũng đi qua $T$.

Thật vậy: Gọi $R,Q,N,M$ lần lượt là trung điểm các đoạn $A_1A_4,A_2A_4,A_3A_4,A_1A_2$.

Theo hệ thức Salơ cho góc định hướng giữa hai đường thẳng ta có:

$(T_{12},TA_{13})\equiv(TA_{12},TR)+(TR,TA_{13})(\text{ mod }\pi)(1)$.

Vì đường tròn ngoại tiếp các tam giác $NRA_{12},QRA_{13}$ tương ứng là các đường tròn Ơle của các tam giác $A_3A_4A_1,A_4A_1A_2$ nên:

$\left\{\begin{array}{I} (TA_{12},TR)\equiv (NA_{12},NR)(\text{ mod }\pi)\\ (TR,TA_{13})\equiv(QR,QA_{13})(\text{ mod }\pi) \end{array}\right.$(2).

Từ $(1),(2)$ suy ra: $(TA_{12},TA_{13})\equiv (NA_{12},NR)+(QR,QA_{13})(\text{ mod }\pi)$.(3)

Vì $NR\parallel A_3A_1,QR\parallel A_2A_1$, nên từ $(3)$ có:

$(TA_{12},TA_{13})\equiv(A_3A_{12},A_3A_1)+(A_2A_1,A_2A_{13})(\text{ mod }\pi)$(4).

Vì bốn điểm $A_1,A_3,A_{14},A_{12}$ cùng thuộc một đường tròn: bốn điểm $A_1,A_2,A_{14},A_{13}$ cùng thuộc một đường tròn nên:

$(A_3A_{12},A_3A_1)\equiv (A_{14}A_{12},A_{14}A_1)(\text{ mod }\pi)$;

$(A_2A_1,A_2A_{13})=(A_{14}A_1,A_{14}A_{13})(\text{ mod } \pi)$.(5)

Từ $(4)(5)$ suy ra:

$(TA_{12},TA_{13})\equiv (A_{14}A_{12},A_{14}A_1)+(A_{14}A_1,A_{14}A_{13})\equiv (A_{14}A_{12},A_{14}A_{13})(\text{ mod }\pi)$.

Vậy: $T$ thuộc đường tròn ngoại tiếp tam giác $A_{12}A_{13}A_{14},$ tức là $T$ thuộc đường tròn $(C_1)$. Tương tự, $T$ thuộc các đường tròn $(C_2),(C_3),(C_4)$. Bài toán được chứng minh.

Lời giải bài 54: Do $f$ tăng thực sự nên: $f(n)<f(n+1)<...<f(2n)=f(n)+n$.

Thành thử: $f(n+1)=f(n)+1$.

Đặt $f(1)=a$ ta có: $f(n)=a+n-1$.

Thay $n=a^2+a+2$ ta suy ta: $f(a^2+a+2)=a+a^2+a+1=(a+1)^2$.

$\implies a^2+a+2$ là số chính phương.

Mặt khác: $a^2<a^2+a+2<(a+2)^2$.

$\implies a^2+a+2=(a+1)^2\implies a=1$.

Vậy $f(n)=n$. Thử lại ta thấy thỏa mãn.


Bài viết đã được chỉnh sửa nội dung bởi tritanngo99: 14-09-2018 - 06:11


#74
tritanngo99

tritanngo99

    Đại úy

  • Điều hành viên THPT
  • 1644 Bài viết

Bài 55: Một dãy $a_1a_2...a_n$ với $a_i\in\left\{0;1\right\}$ gọi là một xâu nhị phân có độ dài $n$. Hỏi có bao nhiêu xâu nhị phân độ dài $n(n\ge 4)$ chứa đúng hai lần xuất hiện của $01$.

Bài 56: Cho dãy số $(x_n)$ được xác định như sau: $x_1=0,x_{n+1}=(\frac{1}{27})^{x_n}$ với mọi $n\in \mathbb{N^*}$. Chứng minh rằng dãy số $(x_n)$ có giới hạn hữu hạn và tìm giới hạn đó.

 


#75
tritanngo99

tritanngo99

    Đại úy

  • Điều hành viên THPT
  • 1644 Bài viết

Lời giải bài 55: Trước hết, ta chứng minh bổ đề sau: 

Bổ đề: Cho $k,n\in \mathbb{N}$. Khi đó số nghiệm nguyên dương của phương trình: $x_1+x_2+...+x_{k}=n(1)$ là $C_{n-1}^{k-1}$.

Thật vậy ứng với mỗi nghiệm $(x_1,x_2,...,x_k)$ ta cho tương ứng với dãy nhị phân.

$\underbrace{1...1}_{x_1}0\underbrace{1...1}_{x_2}...0\underbrace{1...1}_{x_k}$.

trong đó $k-1$ chữ số $0$ được đặt vào $n-1$ vị trí giữa $n$ chữ số $1$. Dễ thấy phép tương ứng song ánh. Vậy số nghiệm nguyên dương của $(1)$ bằng số cách đặt $k-1$ chữ số $0$ vào $n-1$ vị trí giữa $n$ chữ số $1$, do đó bằng $C_{n-1}^{k-1}$. Bổ đề được chứng minh.

Trở lại bài toán: Mỗi xâu nhị phân có chứa đúng hai lần xuất hiện $01$ có dạng:

$ABABA,ABABAB,BABA,BABAB$ trong đó $A$ là xâu nhị phân gồm toàn chữ số $1$; $B$ là xâu nhị phân gồm toán chữ số $0$, mỗi xâu có độ dài ít nhất là $1$.

Xét xâu dạng: $ABABA$: Số xâu có dạng này chính là nghiệm nguyên dương của phương trình: $x_1+x_2+.x_3+x_4+x_5=n$.

Theo bổ đề trên số xâu dạng này là: $C_{n-1}^{4}$.

Tương tự số xâu dạng $ABABAB,BABA,BABAB$ tương ứng là $C_{n-1}^{5},C_{n-1}^{3}$ và $C_{n-1}^{4}$.

Thành thử số xâu thỏa mãm điều kiện đề bài là: $(C_{n-1}^3+C_{n-1}^4)+(C_{n-1}^4+C_{n-1}^5)=C_n^4+C_n^5=C_{n+1}^5(\text{ Dùng }C_{n+1}^{k}=C_{n}^{k}+C_{n}^{k-1})$.

Lời giải bài 56: Nhận xét rằng $x_n\ge 0,\forall n\in \mathbb{N^*}$. Xét hàm số $f(x)=(\frac{1}{27})^{x}$ nghịch biến trong khoảng $[0;+\infty)$. Khi đó $x_{n+1}=f(x_n),\forall n\in\mathbb{N^*}$.

và $f(x)\le f(0)$ nên $0\le x_n\le 1,\forall n\in \mathbb{N^*}$.

Ta có: $x_1=0,x_2=1,x_3=\frac{1}{27}$ nên $x_1\le x_3$ và $x_4=f(x_3)\le f(x_1)=x_2$.

Tiếp theo, ta chứng minh bằng phương pháp quy nạp,$x_{2n-1}\le x_{2n+1}$ và $x_{2n+2}\le x_{2n}\forall n\in \mathbb{N^*}$.

Thật vậy, giả sử có $x_{2n-1}\le x_{2n+1}$ thì

$f(x_{2n-1})\ge f(x_{2n+1})$ nên $x_{2n}\ge x_{2n+2}$ và vì vậy $f(x_{2n})\le f(x_{2n+2})$.

$\implies x_{2n+1}\le x_{2n+3}$.

Tương tự, giả sử có: $x_{2n}\ge x_{2n+2}$ thì:

$f(x_{2n})\le f(x_{2n+2})$ nên $x_{2n+1}\le x_{2n+3}$ và vì vậy $f(x_{2n+1})\ge f(x_{2n+3})$. Suy ra: $x_{2n+2}\ge x_{2n+4}$.

Vậy $(x_{2n-1})$ là dãy đơn điệu giảm và dãy $(x_{2n})$ là dãy đơn điệu tăng và đều thuộc $[0;1]$ nên có giới hạn hữu hạn:

$lim_{n\to+\infty}x_{2n}=a,lim_{n\to+\infty}x_{2n-1}=b$.

và $a=lim_{n\to+\infty}x_{2n+2}=lim_{n\to+\infty}f(x_{2n+1})=lim_{n\to+\infty}f(f(x_{2n}))=f(f(a))$

nên $a=(\frac{1}{27})^{(\frac{1}{27})^a}$. Suy ra: $a=\frac{1}{3}$.

Tương tự ta cũng thu được $b=\frac{1}{3}$. Vậy nên $a=b$ và $lim_{n\to+\infty}x_n=\frac{1}{3}$. 

 

 

Bài viết đã được chỉnh sửa nội dung bởi tritanngo99: 07-09-2018 - 03:50


#76
tritanngo99

tritanngo99

    Đại úy

  • Điều hành viên THPT
  • 1644 Bài viết

Bài 57: Tìm số nguyên tố $p$ nhỏ nhất sao cho $[(3+\sqrt{p})^{2n}]+1$ chia hết cho $2^{n+1}$ với mỗi số tự nhiên $n$, trong đó kí hiệu $[x]$ là phần nguyên của $x$.

Bài 58: Tìm giá trị lớn nhất của biểu thức $x^2(y-z)+y^2(z-y)+z^2(1-z)$ trong đó $x,y,z$ là các số thực thỏa mãn điều kiện $0\le x\le y\le z\le 1$. 



#77
Hr MiSu

Hr MiSu

    Thượng sĩ

  • Thành viên
  • 206 Bài viết

Bài 57: Tìm số nguyên tố $p$ nhỏ nhất sao cho $[(3+\sqrt{p})^{2n}]+1$ chia hết cho $2^{n+1}$ với mỗi số tự nhiên $n$, trong đó kí hiệu $[x]$ là phần nguyên của $x$.

Với $p=2$ thì cho $n=3$ ko thỏa mãn!

Với $p=3$ thì cho $n=1$ ko thỏa mãn!

Với $p=5$ ta chứng minh đây là giá  trị nhỏ nhất thỏa mãn, thật vậy,

Dễ chứng minh: $(3+\sqrt{5})^{2n}+(3-\sqrt{5})^{2n}\in \mathbb{N}$ với mọi n.

Mặt khác $0<(3-\sqrt{5})^{2n}<1$ với mọi $n$ nên $[(3+\sqrt{5})^{2n}]+1=(3+\sqrt{5})^{2n}+(3-\sqrt{5})^{2n}$ với mọi $n$

Mặt khác ta có công thức truy hồi của dãy $(3+\sqrt{5})^{n}+(3-\sqrt{5})^{n}$ là $x_{n+2}-6a_{n+1}+4a_n=0$

từ đó quy nạp ta có $a_{2n}\vdots 2^{n+1}$ với mọi $n$


Bài viết đã được chỉnh sửa nội dung bởi Hr MiSu: 07-09-2018 - 07:33

s2_PADY_s2

Hope is a good thing, maybe the best thing, and no good thing ever dies


#78
tritanngo99

tritanngo99

    Đại úy

  • Điều hành viên THPT
  • 1644 Bài viết

Lời giải bài 57:

+ Với $p=2$ ta chọn $n=2$ thì $[(3+\sqrt{2})^4]+1=378\not\vdots 2^3$.

+ Với $p=3$ ta chọn $n=1$ thì $[(3+\sqrt{3})^2]+1=23\not\vdots 2^2$.

Ta chứng minh $p=5$ thỏa mãn yêu cầu bài toán.

Do $(3+\sqrt{5})^{2n}+(3-\sqrt{5})^{2n}\in \mathbb{Z}$.

và $0<(3-\sqrt{5})^{2n}<1$

nên $(3+\sqrt{5})^{2n}+(3-\sqrt{5})^{2n}-1<(3+\sqrt{5})^{2n}<(3+\sqrt{5})^{2n}+(3-\sqrt{5})^{2n}$.

Thành thử: $[(3+\sqrt{5})^{2n}]+1=(3+\sqrt{5})^{2n}+(3-\sqrt{5})^{2n}$.

Lại có: 

$(3+\sqrt{5})^{2n}+(3-\sqrt{5})^{2n}=(14+6\sqrt{5})^{n}+(14-6\sqrt{5})^n$.

$=2^{n}[(7+3\sqrt{5})^n+(7-3\sqrt{5})^n]=2^{n}.2m=2^{n+1}.m\vdots 2^{n+1}$.

Vậy $p=5$ là số nguyên tố cần tìm.

Lời giải bài 58: 

Ta có: 

$x^2(y-z)+y^2(z-y)+z^2(1-z)\le 0+\frac{1}{2}.y.y.(2z-2y)+z^2(1-z)\le \frac{1}{2}.(\frac{y+y+2z-2y}{3})^3+z^2(1-z)$.

$=z^2(\frac{4}{27}.z+1-z)=z^2(1-\frac{23}{27}z)$.

=$(\frac{54}{23})^2.(\frac{23}{54}z)(\frac{23}{54}z)(1-\frac{23z}{27})$.

$\le (\frac{54}{23})^2.(\frac{1}{3})^3=\frac{108}{529}$.

Đẳng thức xảy ra khi và chỉ khi $\frac{23}{54}z=1-\frac{23}{27}z,y=2z-2y$ và $x=0$, tức là $z=\frac{18}{23},y=\frac{23}{23}$ và $x=0$.

 

 

 

 


#79
tritanngo99

tritanngo99

    Đại úy

  • Điều hành viên THPT
  • 1644 Bài viết

Bài 59: Hỏi có tất cả bao nhiêu đa thức $P_n(x)$ bậc $n$ chẵn thỏa mãn các điều kiện:

1) Các hệ số của $P_n(x)$ thuộc tập hợp $M=\left\{0,-1,1\right\}$ và $P_n(0)\ne 0$.

2) Tồn tại đa thức $Q(x)$ có các hệ số thực thuộc $M$ sao cho $P_n(x)\equiv (x^2-1)Q(x)$.

Bài 60: Cho tứ giác lồi $ABCD$ với các cạnh $AB=a,BC=b,CD=c,DA=d$. Gọi $S$ là diện tích của $ABCD$. Chứng minh rằng với $n\ge 1$ thì $4S^n\le a^{2n}+b^{2n}+c^{2n}+d^{2n}-[(\frac{a-b}{2})^{2n}+(\frac{b-c}{2})^{2n}+(\frac{c-d}{2})^{2n}+(\frac{d-a}{2})^{2n}]$  

 


#80
hozymary

hozymary

    Binh nhì

  • Thành viên mới
  • 19 Bài viết

Bài 60

Với $n\in \mathbb{N}$ thì không có vấn đề gì. Nhưng nếu $n\in \mathbb{R}$ thì cần phải chỉnh đề lại thành:

$$4S^n \le \sum a^{2n} - \sum \left[\left(\frac{a-b}{2}\right)^2\right]^n$$

Áp dụng công thức $S=\frac{1}{2}(ab \sin B+cd\sin D)=\frac{1}{2}(ad\sin A+bc\sin C)$ ta suy ra:

$$S=\frac{ab\sin B+bc\sin C +ca\sin D +da\sin A}{4}\le \frac{ab+bc+cd+da}{4}$$

 

Mặt khác ta có $\frac{a^{2n}+b^{2n}}{2}\ge \left[\left(\frac{a-b}{2}\right)^2\right]^n+\left[\left(\frac{a+b}{2}\right)^2\right]^n\ (1)$. Do đó:

\[\sum a^{2n} - \sum \left[\left(\frac{a-b}{2}\right)^2\right]^n\ge \sum \left(\frac{a+b}{2}\right)^{2n}\ge\sum (ab)^n\]

Áp dụng bất đẳng thức Jensen cho hàm $f(x)=x^n$ (hàm lồi do $n\ge 1$) thì $\frac{\sum (ab)^n}{4}\ge \left(\frac{\sum ab}{4}\right)^n$.

Suy ra $\sum a^{2n} - \sum \left[\left(\frac{a-b}{2}\right)^2\right]^n\ge 4S^n$.

 

Chứng minh (1)






0 người đang xem chủ đề

0 thành viên, 0 khách, 0 thành viên ẩn danh